¿Por qué los puntos lagrangianos L4L4L_4 y L5L5L_5 son estables?

Este diagrama de wikipedia muestra la energía potencial gravitatoria del sistema de dos cuerpos sol-tierra, y demuestra claramente la semiestabilidad del L 1 , L 2 , y L 3 puntos lagrangianos. Las flechas azules indican menor energía potencial, las rojas mayor, por lo que cualquier movimiento en el plano perpendicular a las masas requeriría energía y sin ella, un objeto allí se asentaría.

Puntos de Lagrange

sin embargo, el L 4 y L 5 se afirma que es estable, aunque las flechas de dirección indican que está en la parte superior de un pozo de gravedad, y caería en una energía potencial más baja en cualquier dirección. ¿Qué es lo que hace que estos puntos sean estables sino la gravedad? ¿Qué me estoy perdiendo?

@AlanSE No creo que sea correcto. De acuerdo con todo lo que he encontrado y el OP, las flechas azules indican un gradiente negativo.
Después de ver las respuestas a esta pregunta que atribuyen la estabilidad de L4 y L5 a la fuerza de Coriolis a pesar de la inestabilidad del potencial en esos puntos, los lectores pueden preguntarse si L1, L2 y L3 son estables a pesar de la forma de silla de montar del potencial en esos puntos. . Una respuesta que aborda estos problemas está aquí: 363801/7911 .

Respuestas (9)

Cuando observa la dinámica en el marco de referencia giratorio, hay 4 fuerzas que actúan sobre la partícula: las dos fuerzas gravitatorias de los cuerpos masivos, el empuje centrífugo que se aleja del centro de rotación (ubicado entre los objetos masivos) y la fuerza de Coriolis. .

Las primeras tres fuerzas dependen de la posición de la partícula, y pueden derivarse de un potencial (que también depende de la posición), cuyas curvas de nivel se muestran en la imagen presentada con la pregunta. Este potencial tiene máximos locales en L4 y L5.

La fuerza de Coriolis depende de la velocidad de la partícula: es perpendicular a ella, contenida en el plano de movimiento y proporcional a la velocidad. Curva el movimiento de la partícula hacia la derecha (si los cuerpos masivos y el sistema de referencia giran en sentido antihorario, que es lo que ves en nuestro Sistema Solar si te paras en el polo norte de la Tierra).

Si una partícula colocada en L4 intenta abandonar el punto con una velocidad suave, la fuerza de Coriolis curva su trayectoria. La trayectoria es demasiado ondulada para llegar a alguna parte. Vea la animación en http://demonstrations.wolfram.com/OrbitsAroundTheLagrangePointL4/ .

Por supuesto, esto no prueba que la partícula permanecerá cerca de L4 para siempre. No sé una prueba. He visto algunos cálculos que muestran que la ecuación dinámica linealizada en L4 es estable si la proporción de masa de los objetos masivos es lo suficientemente grande, pero esto tampoco es suficiente para probar la estabilidad en el problema no linealizado.

Estaría convencido de que el equilibrio es estable si se me mostrara que existe una cantidad conservada (dependiendo de la posición y la velocidad) que tiene un extremo local estricto en ese punto del espacio de fase (posición = L4, velocidad = 0).

La "energía" (potencial discutido anteriormente + energía cinética medida en nuestro sistema de referencia no inercial) se conserva porque la fuerza de Coriolis es perpendicular a la trayectoria, por lo que no realiza trabajo (de hecho, en la mecánica de Lagrange se deriva de un potencial que depende de la posición y velocidad de la partícula). Pero esta cantidad no tiene un extremo en nuestro punto de equilibrio, porque el potencial tiene un máximo local en L4 y el término cinético es mínimo cuando la velocidad es 0.

Entonces no puedo probar que el equilibrio es estable.

Aquí hay otra forma de verlo. Dejar METRO 1 , METRO 2 , METRO 3 sean nuestras tres masas. En el problema de los tres cuerpos que estamos considerando, el marco completo que contiene METRO 1 , METRO 2 y METRO 3 está girando

Tiene razón al pensar que si ese marco estuviera fijo, los puntos L4 y L5 no serían estables. Después de todo, si te perturbas METRO 3 desde L4 o L5, entonces debería rodar cuesta abajo.

Pero hay otra fuerza en el trabajo aquí. Debido a que el marco está girando, hay una fuerza ficticia llamada fuerza de Coriolis que sentirías. Esta es la misma fuerza que hace que los huracanes giren en espiral cuando se ven desde el espacio.

Cuando se tiene en cuenta la fuerza de Coriolis, L4 y L5 se convierten en puntos fijos estables. Es decir, si perturbas METRO 3 desde L4 un poco, simplemente permanecerá en su distancia perturbada y orbitará el punto L4.

Si alguien quiere ver las matemáticas para esto, consulte este útil artículo.

Esto es útil. Wikipedia da Coriolis, centrífuga y la fuerza de Euler como las fuerzas "ficticias" para reformular un problema a un marco de referencia giratorio. No estoy seguro de cómo lo atribuiste solo al coriolis.
@AlanSE, la fuerza de Euler es cero, porque la aceleración de rotación del marco es cero y la fuerza centrífuga ya se tiene en cuenta como potencial en el problema original. Eso deja a Coriolis como la única opción.
Solo L1 es estable en un marco que no gira, pero puedo entender claramente L2 y L3 en el contexto de la rotación. L4 y L5 no parecen intuitivamente estables y estoy buscando una explicación intuitiva.
@Ehyrk No creo que haya una explicación más intuitiva que esta. Podría ser útil enfatizar que la estabilidad de L4 y L5 no significa que la masa retroceda hasta el punto. De hecho, es un concepto llamado estabilidad de Lyapunov, lo que significa que la masa permanecería para siempre cerca de L4 o L5. Si no es inmediatamente intuitivo que un marco giratorio puede tener este efecto, entonces necesita desarrollar su intuición para el tema. Un google rápido arroja muchos recursos. ¡Espero que ayude!
@Ehyrk: L1 es el único punto de equilibrio en un marco no giratorio, pero es un equilibrio inestable. Si un cuerpo se mueve progresivamente desde L1 hacia el sol o la tierra, adiós cuerpo...

Como han comentado otros, incluido OP, el potencial efectivo (que consiste en la gravedad y el potencial centrífugo )

(1) V   =   GRAMO metro 1 | z 1 | GRAMO metro 2 | z 2 | Ω 2 | z | 2 2
en el plano orbital R 2 C tiene un máximo global en los puntos de Lagrange L 4 y L 5 ,
(2) z 1   =   R Exp { ± i π 3 }   =   R 2 ± 3 i R 2 , z 2   =   R Exp { i π 3 }   =   R 2 ± 3 i R 2 .
Estoy aquí usando la misma notación que mi respuesta Phys.SE aquí :
(3) z 1   :=   z r 1     0 , z 2   :=   z r 2     0 , Ω 2   :=   GRAMO ( metro 1 + metro 2 ) R 3 .
¡Por lo general, una partícula de prueba no quiere estar en un máximo global! Sin embargo, no debemos olvidarnos de la fuerza de Coriolis .

Declaración principal. Cuando la partícula de prueba intenta salir L 4 o L 5 , la fuerza de Coriolis evitará esto a través de la desviación si una de las relaciones de masa metro 1 / metro 2 o metro 2 / metro 1 excede

(4) 25 2 + 3 2 69     25

En esta respuesta, me gustaría calcular esta condición de relación de masa (4), que también se menciona en Wikipedia .

El hessiano del potencial efectivo (1) es

(5) H i k   :=   2 V z i z k , i , k     { 1 , 2 } , z     R 2 , H   =   GRAMO metro 1 ( 3 z 1 z 1 T | z 1 | 5 1 2 × 2 | z 1 | 3 ) GRAMO metro 2 ( 3 z 2 z 2 T | z 2 | 5 1 2 × 2 | z 2 | 3 ) Ω 2 1 2 × 2 .
A L 4 o L 5 , la arpillera se convierte en
(6) H   =   3 Ω 2 4 ( 1 ± 3 ϵ 12 ± 3 ϵ 12 3 ) , ϵ 12   :=   ϵ 1 ϵ 2   =   metro 1 metro 2 metro 1 + metro 2   =   1 2 ϵ 2     ] 1 , 1 [ .
La traza y el determinante son
(7) t r ( H )   =   3 Ω 2 y det ( H )   =   27 Ω 4 dieciséis ( 1 ϵ 12 2 )   >   0.

Ahora usamos el siguiente teorema 1 mencionado en la ref. 1:

Teorema. Que se dé una arpillera H para un potencial efectivo V : R 2 R en un punto de Lagrange. Dejar Ω Sea la velocidad angular. Las siguientes 3 condiciones son condiciones necesarias y suficientes para que el punto de Lagrange sea estable:

  1. C   :=   det ( H )     0.

  2. B   :=   t r ( H ) + 4 Ω 2     0.

  3. D   :=   B 2 4 C     0.

NB: El teorema anterior ignora los términos de orden superior en V , que podría volverse importante si det ( H ) = 0 .

Se cumplen las dos primeras condiciones:

(8) C   =   27 Ω 4 dieciséis ( 1 ϵ 12 2 )   >   0 y B   =   Ω 2   >   0.
La tercera condición dice
(9) 0   <   D   =   27 ϵ 12 2 23 4   =   27 ( ϵ 2 2 ϵ 2 + 1 27 ) .
Las raíces de la ecuación cuadrática D = 0 son
(10) metro 2 metro 1 + metro 2     ϵ 2   =   1 2 ± 69 18     { 25 / 26 , 1 / 26
Esto conduce a la condición (4).

Referencias:

  1. J. Binney & S. Tremaine, Galactic Dynamics, 2ª edición (2008); pags. 181-182.

--

1 Prueba del teorema: la EOM linealizada en un punto de Lagrange z = 0 en el plano orbital R 2 lee

(11) z ¨     2 Ω   j z ˙ Coriolis H z , j   :=   ( 0 1 1 0 ) ,

donde el primer término de la derecha. es la fuerza de Coriolis. La arpillera es una matriz simétrica real y, por lo tanto, diagonalizable con 2 ejes principales. Después de una posible rotación de coordenadas z mi j ϕ z , podemos suponer que la arpillera

(12) H   =   ( H 1 0 0 H 2 )

es diagonal. (¡La rotación conmuta con el término de Coriolis (11) y deja invariantes las 3 condiciones en el teorema!) Los EOM (11) son 2 ODE homogéneas de segundo orden acopladas con coeficientes constantes. Su ecuación característica es una ecuación de cuarto orden.

(13) 0   =   | λ 2 + H 1 2 Ω λ 2 Ω λ λ 2 + H 2 |   =   ( λ 2 ) 2 + B λ 2 + C ,
que tiene 4 raices
(14) λ 2   =   B ± D 2 .
ecuación (13) es una ecuación de segundo orden. en λ 2 . La solución a las 2 coordenadas z 1 y z 2 son combinaciones lineales de exponenciales mi λ t , dónde λ es una raíz. La condición de estabilidad es que R mi ( λ ) 0 para las 4 raíces.

Sin embargo, debido a la simetría en la ec. (13), si λ es una raíz, también lo es λ . Entonces la condición de estabilidad es que R mi ( λ ) = 0 para las 4 raíces, es decir, que λ es imaginario. O equivalentemente, que λ 2 0 no es positivo para las 4 raíces.

Esto sólo es posible si el discriminante D 0 en la ec. (14) no es negativa, es decir, la tercera condición del teorema.

Las condiciones 1 y 2

(15) C , B     0 λ ± 2     0
se sigue del hecho bien conocido de que
(dieciséis) C   =   λ + 2 λ 2 y B   =   λ + 2 + λ 2
en la ec. de segundo orden. (13) es el producto y la suma de sus raíces, respectivamente.

Creo que hay algunas buenas explicaciones aquí, pero intentaré agregar una explicación muy simplificada y, con suerte, intuitiva.

Primero, pongámonos de acuerdo en algunas convenciones. Supongamos que la Tierra gira en sentido antihorario alrededor del Sol. Va en una órbita circular a una distancia R (desde el baricentro) con velocidad angular Ω .

Nos vamos a centrar en L4. Primero suponga que comenzó exactamente en L4, moviéndose a la velocidad adecuada para permanecer allí. Este es ciertamente un equilibrio que no requiere ninguna fuerza para mantenerlo; dejaremos el tema de la estabilidad solo por un momento. ¿A qué velocidad vas mientras permaneces en L4? Bueno, su velocidad angular debe ser simplemente la velocidad patrón Ω , lo que significa que en el marco corrotatorio (aquel en el que el Sol y la Tierra están fijos tal como se muestra en la imagen) no se mueve en absoluto. En el marco inercial con el baricentro del sistema en reposo, debe tener una velocidad Ω R , y está dirigido completamente en la dirección azimutal, es decir, tangente al círculo de su órbita.

Ahora suponga que está desplazado de esa acogedora posición de equilibrio. Para ser más específicos, supondremos que ahora se encuentra un poco más cerca del baricentro; te movieron a lo largo de la línea que lo conecta con L4. Además, no perdió demasiada energía cinética (para ser preciso en el siguiente párrafo). Esto es importante. Si algo, digamos un arrastre viscoso, sigue drenando su energía total, tendrá que caer en una parte más baja de la superficie potencial y no podrá permanecer cerca de L4.

Otra suposición simplificadora: tu impulso sigue en la misma dirección que cuando estabas felizmente sentado en L4. Es decir, es enteramente azimutal. Entonces, ¿cuál es tu situación ahora? Bueno, tu nueva distancia desde el baricentro es R Δ R . Para mantener la velocidad del patrón (es decir, para no moverse con respecto a la Tierra y L4 en ese diagrama), necesita una velocidad azimutal de Ω ( R Δ R ) < Ω R . No necesita tanta velocidad y, por lo tanto, no necesita tanta energía cinética para mantenerse al día con la rotación. Originalmente tenía una energía cinética específica (energía cinética por unidad de masa) de ( 1 / 2 ) Ω 2 R 2 , y ahora mientras tengas más de ( 1 / 2 ) Ω 2 ( R Δ R ) 2 , avanzará por delante del patrón .

Entonces, en referencia a la figura, se desplazó a algún lugar debajo ya la izquierda de L4, y ahora se moverá hacia arriba y hacia la izquierda, al menos inicialmente. Al igual que cualquier masa de prueba con una velocidad tangencial demasiado grande para una órbita circular, te estarás moviendo hacia afuera. En el marco giratorio, esto podría interpretarse como una manifestación de la fuerza de Coriolis. [Aparte: La fuerza de Coriolis actúa en la dirección Ω × v , y con Ω apuntando hacia fuera de la página y v (su velocidad en el marco giratorio ) apuntando azimutalmente en sentido antihorario, puede ver que esto causará una aceleración dirigida hacia afuera]. Pero esto es más obvio en el marco inercial, donde ahora va demasiado rápido para que la gravedad del Sol doble su camino en un círculo.

¿Así que lo que ocurre? A medida que avanza por delante del patrón (su velocidad angular es mayor que Ω ), también eres empujado hacia afuera. Eventualmente te encuentras a distancia R de nuevo, pero ahora está por delante (en sentido contrario a las agujas del reloj, arriba ya la izquierda de) L4. La "fuerza" radial se desvanecerá en algún lugar por aquí, pero recuerda que tienes inercia, ¡ya sea que tu marco de referencia sea inercial o no! Así que te deslizarás hacia afuera, a distancias del baricentro mayores que R . De este lado de R Sin embargo, todo se invierte. No tienes suficiente velocidad para mantener una velocidad angular Ω , entonces te quedas atrás. Ahora se está moviendo en sentido contrario a las agujas del reloj en ese marco giratorio, girando alrededor del exterior de L4. Por supuesto, debido a que vas demasiado lento para mantener una órbita circular (descrito en el marco de inercia), o porque hay una fuerza de Coriolis dirigida hacia adentro (descrita en el marco giratorio), estás acelerado hacia adentro. Eventualmente, se moverá detrás de L4 y regresará a algún lugar cerca de donde comenzó este viaje justo después de ser desplazado.

Las órbitas pueden ser complicadas de describir analíticamente, pero es de esperar que esto muestre dónde entran en juego las "fuerzas" y hacia dónde se dirigen. Terminas trazando un camino que va en el sentido de las agujas del reloj alrededor de L4 (nota que todos mis otros usos de "sentido horario"/"sentido antihorario" han sido con respecto al baricentro).

Una nota final: en el marco giratorio, este es realmente un problema de dinámica , en el que no se puede obtener el movimiento simplemente diferenciando un potencial simple. Tenga en cuenta que empezamos a hacer eso, y eso es lo que lleva a la confusión. Además de los potenciales gravitatorios inducidos por las dos masas, agregamos un término al potencial efectivo (la función cuyos contornos se trazan en el diagrama) para tener en cuenta la fuerza centrífuga:

Φ efecto ( X ) = GRAMO ( METRO | X X | + METRO | X X | + METRO + METRO 2 | X X | 3 | X | 2 ) ,
dónde X es el vector bidimensional en el plano del sistema con origen en el baricentro. ( Galactic Dynamics de Binney & Tremaine deriva esto en un rodeo aparte; probablemente haya tratamientos más directos). Sin embargo, solo podemos detenernos aquí si solo queremos considerar las fuerzas sobre las partículas en reposo en el marco giratorio. Como se señaló en otra publicación, la fuerza centrífuga ( Ω × Ω × X ) no es la única fuerza ficticia de la que tenemos que preocuparnos. Si el objeto de prueba tiene una velocidad con respecto al marco giratorio, como en el caso que nos ocupa, tenemos la fuerza de Coriolis ( 2 Ω × X ˙ ), que no tomamos en cuenta al establecer el potencial efectivo. La razón es que no es solo una función del espacio de posición 2-D, sino de todo el espacio de fase 4-D. Hay otra fuerza ficticia común que afortunadamente no tuvimos que considerar: la fuerza de Euler ( Ω ˙ × X ). Esto solo entraría en juego si la velocidad del patrón cambiara (por ejemplo, si la Tierra tuviera alguna aceleración adicional, tal vez por estar en una órbita excéntrica).

Tres temas:

  1. Hay una analogía electromagnética para las fuerzas no inerciales en un marco de referencia giratorio.
  2. Antes de ver el problema restringido de tres cuerpos de la pregunta, analice el problema familiar de dos cuerpos en un marco de referencia giratorio. Creo que es instructivo interpretar estos resultados en términos de las soluciones conocidas para este problema.
  3. Finalmente, mira los puntos de Lagrange.

1. Analogía electromagnética. Resulta que las fuerzas no inerciales sobre una masa metro en un marco de referencia giratorio se puede manipular exactamente en la forma de la fuerza electromagnética de Lorentz sobre una carga q en un marco inercial.

Referencia de Moreno y Barrachina (pdf) . Hay algunos errores tipográficos desafortunados, pero creo que los resultados que afirmo son precisos.

Específicamente, para un marco de referencia giratorio con velocidad angular ω , si el potencial escalar electromagnético ϕ y vector potencial A se toman como:

ϕ = ( metro q ) ( ω × r ) 2 2
A = ( metro q ) ω × r
entonces la fuerza electromagnética sobre la carga q en un marco inercial es lo mismo que las fuerzas no inerciales en la masa metro en el marco giratorio.

En particular, para una velocidad angular constante ω que es perpendicular al plano de movimiento (como en este problema), en coordenadas cilíndricas ( r , θ , z ) , el campo eléctrico análogo mi y campo magnético B son:

mi = ( metro q ) ω 2 r r ^
B = ( metro q ) 2 ω z ^

El campo eléctrico "simula" la fuerza centrífuga y el campo magnético la fuerza de Coriolis. Toda la intuición de la fuerza de Lorentz continúa: la fuerza de Coriolis desvía un cuerpo perpendicular a su velocidad, formando círculos en ausencia de otras fuerzas.

2. Problema de dos cuerpos en un marco giratorio. El problema de los dos cuerpos ( METRO s (sol), METRO mi (tierra)) se puede formular en un marco giratorio con velocidad angular ω (haciendo de la velocidad angular la variable independiente). Realización de la conversión habitual a un potencial de un solo cuerpo tu , se obtiene:

tu = GRAMO METRO t METRO r r METRO r ω 2 r 2 2 METRO r ω r 2 θ ˙
 dónde  METRO t = METRO s + METRO mi , 1 METRO r = 1 METRO s + 1 METRO mi , α = METRO mi METRO t , β = METRO s METRO t

Resolviendo para un punto estacionario r o (con θ ˙ = 0 ), se obtiene el resultado habitual:

r o = ( GRAMO METRO t ω 2 ) 1 3 , coordenadas xy del sol  ( X s , y s ) = ( α r o , 0 ) , tierra  ( X mi , y mi ) = ( β r o , 0 )

Pero observe que el potencial en r o es un máximo, no un mínimo!

Para ver lo que sucede, encuentre las ecuaciones de movimiento linealizadas alrededor del punto estacionario.

( X , y ) = ( X o + d X , y o + d y ) , ( v X , v y ) = ( d v X , d v y )
d d t ( d X d y d v X d v y ) = ( 0 0 1 0 0 0 0 1 q X X q X y 0 2 ω q X y q y y 2 ω 0 ) ( d X d y d v X d v y )
 con  q X X = 1 metro d 2 tu d X 2 , q y y = 1 metro d 2 tu d y 2 ,  y  q X y = 1 metro d 2 tu d X d y
Las frecuencias naturales s i son las raíces del polinomio característico, que resulta ser:
s 4 Γ s 2 + Δ = 0
 dónde  Γ = ( q X X + q y y + 4 ω 2 )  y  Δ = q X X q y y q X y 2

Para este caso, sólo q X X = 3 ω 2 es distinto de cero (curvatura negativa en un máximo), por lo que uno encuentra Γ = ω 2 y Δ = 0 , y las frecuencias naturales son:

s 1 , 2 = 0 , s 3 , 4 = ± i ω
correspondiente a la respuesta neutra o al movimiento oscilatorio a la frecuencia de rotación. La fuerza de Coriolis dobla las trayectorias y, por lo tanto, "convierte" un equilibrio inestable en un centro de movimiento oscilatorio.

Interpretación: Una órbita circular (correspondiente al punto estacionario) es solo una posible solución del problema de los dos cuerpos; en general, las órbitas son elipses, por lo que se debe esperar un movimiento oscilatorio alrededor del punto estacionario, periódico a la frecuencia de rotación. (Creo que las 0 frecuencias naturales corresponden a un cambio de posición azimutal). El punto es que el movimiento no es inestable, a pesar de ser un máximo local, y la falta de amortiguamiento (parte real negativa de las frecuencias naturales) debe esperarse .

(Por cierto, si q X X fueran positivas, correspondientes a un mínimo local, las frecuencias naturales oscilatorias resultantes estarían a una frecuencia más alta que ω , y el movimiento resultante no sería elíptico.)

3. Puntos de Lagrange El problema restringido de los tres cuerpos puede analizarse de la misma manera que antes y determinarse la estabilidad de cada punto estacionario. Una referencia es Cornish (pdf) . Resulta que L1, L2 y L3 tienen frecuencias naturales reales positivas y, por lo tanto, son inestables. L4 y L5 tienen todas las frecuencias naturales imaginarias puras (siempre y cuando METRO mi < 25 METRO s ), y por lo tanto tienen movimientos similares a los del problema de dos cuerpos.

Actualización : aquí hay algunas notas sobre el carácter de la dinámica en los diversos puntos de Lagrange:

L4 y L5: estos puntos se parecen más al problema de los dos cuerpos, con una curvatura potencial negativa (desestabilizadora) relativamente pequeña que se estabiliza mediante la fuerza de Coriolis.

L3: Este punto (planeta "x" al otro lado del sol de la tierra) es el más interesante para mí. Dado que las curvaturas son nuevamente pequeñas (como en el problema de los dos cuerpos de la Tierra), y una de ellas es de hecho positiva (estabilizadora), parece extraño que L3 sea inestable. La resolución es que las curvaturas de signo opuesto forman un punto de silla. Un cuerpo puede escapar de tal punto viajando lentamente cerca de la línea de curvatura cero (y por lo tanto de fuerza cero); con el ángulo y la velocidad adecuados, acelera lentamente y la fuerza de Coriolis es ineficaz para detener el escape.

L1 y L2: estos puntos son inestables por dos razones: a) la gran curvatura potencial negativa (desestabilizadora) de la tierra cercana supera el efecto estabilizador de la fuerza de Coriolis y b) las curvaturas principales difieren en signo (como L3), formando un punto de silla.

No estoy seguro de que haya una explicación intuitiva simple, pero intentemos esto:

Antes de que podamos aclarar la razón por la cual estos puntos son estables, tenga en cuenta lo siguiente: si tiene un objeto orbitando alrededor de otro (para hacerlo simple) mucho más grande. Este sistema está aislado y contiene cierta energía total. Ahora, si quisiéramos encontrar otras configuraciones de este sistema de dos cuerpos con la misma energía, las encontraríamos al disminuir la energía potencial (disminuyendo así la distancia entre los dos objetos) en la misma cantidad que aumentamos la energía cinética (por aumentando la velocidad tangencial), dejando así la energía total del sistema sin cambios. Tenga en cuenta que también podríamos hacer lo contrario. Entonces, en resumen: para una energía total dada, el cuerpo en órbita puede estar cerca del objeto central y tener una velocidad tangencial alta, o estar lejos con una velocidad tangencial más baja (ver Keppler'

Volvamos al problema de los tres cuerpos. Para comprender por qué la pequeña masa se encuentra en un estado de equilibrio estable, puede dibujar un triángulo equilátero o mirar el satélite en esta imagen :https://upload.wikimedia.org/wikipedia/commons/thumb/7/78/L4_diagram.svg/683px-L4_diagram.svg.pngDado que el centro de masa (alrededor del cual orbita cada uno de los objetos) NO está en el centro de la masa muy grande, la órbita de la tercera masa (la luna) está ligeramente dentro de la órbita de la masa pequeña (en la imagen, el la órbita de la luna está ligeramente dentro de la órbita del satélite). Si ahora movemos el satélite a lo largo de su trayectoria hacia la luna, no modificamos su distancia radial y por lo tanto no cambiamos su velocidad tangencial (según lo mencionado anteriormente). Lo que hacemos es acercarlo a la luna Y alejarlo de la tierra. Dado que el satélite gira alrededor de la luna (también en el sentido de las agujas del reloj, para tener una idea de eso, imagínese en la luna y trate de averiguar cómo vería el satélite). Al acercarlo, disminuimos la energía potencial entre la luna y la luna. satélite, lo que conduce (ya que la conservación de la energía) a un aumento en la velocidad tangencial (en el sentido de las agujas del reloj alrededor de la luna), por lo que el satélite tiende a alejarse de la tierra (y del centro de masa). Al alejarnos de la tierra aumentamos su energía potencial con respecto a la tierra, lo que hace que gire más lento alrededor de la tierra, por lo que tiende a alejarse de la luna (ya que la rotación que describe alrededor de la tierra también es en el sentido de las agujas del reloj. Al moverse Alejándonos de la luna, aumentamos la energía potencial entre satélite-luna, por lo tanto, disminuimos la velocidad angular.El satélite tiende a moverse hacia la tierra, disminuyendo la energía potencial en el sistema satélite-tierra, por lo tanto, el aumento de la velocidad angular lo acerca a la luna,... y volvemos al principio. por tanto, el satélite tiende a alejarse de la tierra (y del centro de masa). Al alejarnos de la tierra aumentamos su energía potencial con respecto a la tierra, lo que hace que gire más lento alrededor de la tierra, por lo que tiende a alejarse de la luna (ya que la rotación que describe alrededor de la tierra también es en el sentido de las agujas del reloj. Al moverse Alejándonos de la luna, aumentamos la energía potencial entre satélite-luna, por lo tanto, disminuimos la velocidad angular.El satélite tiende a moverse hacia la tierra, disminuyendo la energía potencial en el sistema satélite-tierra, por lo tanto, el aumento de la velocidad angular lo acerca a la luna,... y volvemos al principio. por tanto, el satélite tiende a alejarse de la tierra (y del centro de masa). Al alejarnos de la tierra aumentamos su energía potencial con respecto a la tierra, lo que hace que gire más lento alrededor de la tierra, por lo que tiende a alejarse de la luna (ya que la rotación que describe alrededor de la tierra también es en el sentido de las agujas del reloj. Al moverse Alejándonos de la luna, aumentamos la energía potencial entre satélite-luna, por lo tanto, disminuimos la velocidad angular.El satélite tiende a moverse hacia la tierra, disminuyendo la energía potencial en el sistema satélite-tierra, por lo tanto, el aumento de la velocidad angular lo acerca a la luna,... y volvemos al principio.

Entonces, siguiendo esta línea de pensamiento, podría quedar más claro por qué estos puntos son estables (por cierto, no depende de la dirección de la rotación, trate de pensarlo con una rotación en sentido contrario a las agujas del reloj). Desafortunadamente, este no es el final de la historia: podrías hacer el mismo Gedankenexperiment independiente de las masas reales de la luna y la tierra y siempre encontrarías un punto estable. Lo que realmente es importante, es la relación de masa entre las dos masas. Es solo en un rango bastante pequeño, donde el punto es estable. Para muchos sistemas, el cambio en la distancia entre el satélite y la luna no puede ser compensado por el cambio resultante en la velocidad tangencial (lo que lleva a un cambio compensatorio en la distancia entre el satélite y la tierra) y el satélite dejaría el punto y el éter comenzaría a orbitar alrededor de la luna o el sol. Bien,Esta es una buena descripción del fenómeno.

Espero que haya ayudado.

Por cierto: Tenga en cuenta que la fuerza de Coriolis NO existe. Debe introducirse cuando se utiliza un sistema de referencia acelerado, para compensar la aceleración del sistema de referencia. Por lo general, necesitaríamos introducir esta fuerza ficticia al elegir la Tierra o la Luna como sistema de referencia (ambos están acelerados ya que giran uno alrededor del otro). Recomiendo no comenzar a incluir tales fuerzas inexistentes, ya que podría arriesgarse a confundirlos con los reales. Utilice el centro de masa como sistema de referencia y describa el sistema con el uso de coordenadas esféricas. Al derivar el vector unitario, verá términos "extraños" que deben incluirse para la aceleración. Cuando se usa la segunda ley de Newton, estas termias permanecen del lado de la aceleración y no hay necesidad de inventar nuevas fuerzas.

Como sé, en L4, L5, el potencial del poder gravitacional es máximo, aunque es inusual para la cinemática, que considera que los puntos estables son cuando U->Min, pero en los sistemas dinámicos, los puntos estables pueden ser incluso cuando U->Max, pero luego lo llamamos "Equilibrio dinámico" en el sentido de que el objeto realmente se moverá alrededor del punto estable (pero nunca llegará al punto estable en sí mismo, incluso después de pasar un tiempo infinito) (también hay otras diferencias de los puntos estables clásicos). Matemáticamente, esto se puede probar utilizando la definición de estabilidad de Lyapunov para sistemas dinámicos.

PD Nunca hice eso solo, pero nuestro profesor habló sobre este tema durante nuestro curso de Astrofísica.

Creo que lo que te confunde es el marco de referencia giratorio... Los puntos L4 y L5 giran alrededor del centro de masa (COM) del sistema Tierra-Sol a la misma velocidad que la Tierra y el Sol orbitan uno alrededor del otro. Entonces, cuando haces el experimento mental de lo que le sucede a una masa puntual en L5 o L4 si se desplaza infinitesimalmente, es más fácil pensar en un marco de referencia inercial. Si en la imagen de arriba la Tierra gira en sentido contrario a las agujas del reloj, esta es una descripción cualitativa de lo que le sucede a una masa puntual que se mueve ligeramente desde L5:

  • si se alejara o acercara un poco al COM, no tendría suficiente/tendría demasiada velocidad tangencial para su nueva posición, por lo que comenzaría a quedarse atrás/adelantarse a L5, es decir, moviéndose en sentido horario/antihorario.
  • si se desplaza ligeramente por delante o por detrás de L5, es decir, en sentido antihorario o horario alrededor del COM, su velocidad ya no será tangencial a su órbita, por lo que comenzará a alejarse o acercarse al COM.

Combine esto y obtendrá que un objeto desplazado de su posición en L5 entrará en una órbita en el sentido de las agujas del reloj alrededor de L5. Esta órbita es estable si la relación de las masas es mayor que ~25.

Creo que el hecho de que te estés olvidando (lo cual es fácil, ¡solo leí el artículo para encontrarlo!) es que en una órbita, el momento angular se conserva. Así que si un objeto en L 4 o L 5 se perturba hacia adentro, digamos, luego se acelera ligeramente y luego cruza la órbita. Pero luego se ralentiza de nuevo. En el marco de referencia giratorio, hay algo de movimiento sobre el punto de Lagrange pero representa movimiento sobre el punto estable.

A diferencia de los puntos lagrangianos colineales, los puntos triangulares (L4 y L5) son equilibrios estables (cf. atractor), siempre que la relación de METRO 1 / METRO 2 es mayor que 24.96 . Este es el caso del sistema Sol-Tierra, el sistema Sol-Júpiter y, por un margen menor, el sistema Tierra-Luna. Cuando se perturba un cuerpo en estos puntos, se aleja del punto, pero el factor opuesto al que aumenta o disminuye por la perturbación (ya sea la gravedad o la velocidad inducida por el momento angular) también aumentará o disminuirá, desviando la trayectoria del objeto. en una órbita estable con forma de frijol alrededor del punto (como se ve en el marco de referencia giratorio).

Lo leí, pero no lo entiendo. ¿Puedes explicarlo mejor?
En realidad, el momento angular NO SE conserva, consulte aquí: en.wikipedia.org/wiki/…